Is this limit actually solvable? Reddit calculus problem r/askmath

Поділитися
Вставка
  • Опубліковано 28 вер 2024
  • Is this limit actually solvable? This is a classic calculus limit problem that I saw on Reddit. Subscribe to ‪@bprpcalculusbasics‬ for more calculus tutorials. Also, I am on Reddit as bprp_reddit and you can follow me here: / g0xskgcnue
    Here's the Reddit post on r/askmath: / gk5yqgxzta
    -----------------------------
    Support this channel and get my calculus notes on Patreon: 👉
    / blackpenredpen
    Get the coolest math shirts from my Amazon store 👉 amzn.to/3qBeuw6
    -----------------------------
    #calculus #bprpcalculus #apcalculus #tutorial #math

КОМЕНТАРІ • 214

  • @bprpcalculusbasics
    @bprpcalculusbasics  10 місяців тому +23

    Try this limit next: (L’H rule won’t help!) 👇
    ua-cam.com/video/mMFJUZAHhf0/v-deo.html

  • @steamunlocked21
    @steamunlocked21 10 місяців тому +1102

    Instead of L' Hôpital's Rule, we can just let h=1/a, see that as a -> infinity, h -> 0 and then get limit (x^h-1)/h as h -> 0, which equals lnx

    • @debtanaysarkar9744
      @debtanaysarkar9744 10 місяців тому +74

      Yea, that's how I solved it too😀😀

    • @LinaWainwright
      @LinaWainwright 10 місяців тому +40

      The lim as h->0+ is the definition of the derivative from the right of f(y)=x^y.

    • @dashingmlg601
      @dashingmlg601 10 місяців тому +29

      Uhhh. Wouldn't that give you the same indeterminate value of 0/0 after plugging in h=0. What am i missing?

    • @healer1461
      @healer1461 10 місяців тому +29

      ​@@dashingmlg601Well there are many equivalent ways to define the natural log, and consequently the log for any other base, the historical way is by considering the function: 1 divided by t integrated with respect to t, bounds running from 1 to x, giving it a special name and calling it a day, though with some simple manipulation you can show it will be equal to the limit above. Another equally common way is to note that in the expression of deducting the derivative of an arbitrary exponential you always get the exponential itself times this limit, and since it only depends on the base you choose it must then be a constant. I find the latter more sensible, however you do need to exercise some analysis muscles to show it's convergence and stuff, otherwise it just feels a bit of handwaved definition.

    • @alekhon3900
      @alekhon3900 10 місяців тому +27

      ​@@dashingmlg601(x^h - 1)/h -> lnx as h -> 0 is well-known limit that comes from (e^h - 1)/h -> 1 as h -> 0 (and it comes from ln(x+1)/x -> 1 as h -> 0, that is just logarithmed (1 + h)^(1/h)). So, we don't need L'Hôpital's rule to eliminate this indeterminance

  • @lolok6439
    @lolok6439 10 місяців тому +871

    whenever you have any calculus question which you have no idea how to solve, just answer lnx or e^x and theres gonna be a 90% chance your answer is at least half correct

    • @bprpcalculusbasics
      @bprpcalculusbasics  10 місяців тому +324

      And π. 😆

    • @magnusmalmborn8665
      @magnusmalmborn8665 10 місяців тому +74

      There's also sqrt(2) and 1 or 0, but in mathematics showing how you get the result is way more important than the result itself.

    • @DatBoi_TheGudBIAS
      @DatBoi_TheGudBIAS 10 місяців тому

      @@bprpcalculusbasicsand 1/e

    • @bombintheseeinq
      @bombintheseeinq 10 місяців тому +42

      and the current year for competitive math haha

    • @Trollllium
      @Trollllium 10 місяців тому +21

      @@bprpcalculusbasicsdang Gaussian integrals haha, always makes π appear outa nowhere

  • @LiteralSwan
    @LiteralSwan 10 місяців тому +89

    I like limits like these that have only algebraic operators but have a transcendental value, in this case, a function with transcendental values unless the input itself is a transcendental number (or 1).

    • @DeJay7
      @DeJay7 7 місяців тому

      Yeah it's really quite funny. Getting non-algebraic results with only algebraic operators. But x^(1/a) for any real number a is borderline (if at all) an algebraic operation in this case, so y'know.

    • @LiteralSwan
      @LiteralSwan 7 місяців тому

      @@DeJay7 how so? x^y where x and y are rational numbers is still algebraic, no?

    • @DeJay7
      @DeJay7 7 місяців тому

      @@LiteralSwan It kinda depends on what you mean by algebraic. An expression with a variable on the exponent is NOT considered an algebraic function (Euler's word). Also, if you have b^x and x is irrational and b is any real number not 0 or 1, then at least one of b, x or b^x is transcendental.
      Source: en.wikipedia.org/wiki/Exponentiation

    • @LiteralSwan
      @LiteralSwan 7 місяців тому

      @@DeJay7 hence why i said x and y are rational numbers...

  • @concretemathematics414
    @concretemathematics414 10 місяців тому +1

    i love the absolute tons of expo marker boxes in the background

  • @blueberrypi8686
    @blueberrypi8686 9 місяців тому +2

    It's cool to find out that I am the friend in question.

  • @vincent.0705
    @vincent.0705 10 місяців тому +20

    Hi bprp!! Great video as always. I want to ask do you happen to have any videos on Rolle's theorem?

  • @frtzkng
    @frtzkng 9 місяців тому +1

    I just put it into Grapher, tried out higher and higher values for a, and saw that y=1 at x=e, so this limit approaches the inverse of e^x, which is ln(x). Actually the limit which approaches e^x for a→∞ looks similar, it's (1+(x/a))^a

  • @Ploofles
    @Ploofles 10 місяців тому +13

    I solved it! before watching the video! I feel very smart thank you

    • @blank3580
      @blank3580 10 місяців тому +1

      same btw its easy ques

    • @btb2954
      @btb2954 10 місяців тому +1

      same btw its hard ques

    • @blank3580
      @blank3580 10 місяців тому +1

      @@btb2954 no its not hard

    • @zachansen8293
      @zachansen8293 10 місяців тому

      @@blank3580pretending like there is an objective answer to whether something is easy or hard is just silly. don't be silly.

  • @Temari_Virus
    @Temari_Virus 9 місяців тому +1

    My approach:
    First I wanted to see if the limit even converges, so I took the derivative with respect to x:
    x^((1 - a) / a)
    As a approaches infinity, I think it's quite obvious that this becomes:
    1 / x
    So I'm pretty sure it converges, and I just need to integrate to get the answer:
    a(x^(1 / a)) - a = ln(x) + C
    Plugging in x = 1,
    a(1) - a = C
    C = 0
    Hence, the limit converges to ln(x)

    • @Jonathan-oh8rr
      @Jonathan-oh8rr 9 місяців тому

      I did that (∞√(x))= 1 so you get a(1)-a which equals a-a which equals 0 so lim(a->∞) a(a√x)-a= ∞(∞√x)-∞=∞-∞ and because we know the infinites are the same number we subtract them getting 0

  • @torgegraner7629
    @torgegraner7629 10 місяців тому +1

    Write x^(1/a)-1=e^(ln(x)/a)-1=ln(x)/a+(ln(x) /a)^2/2+..., and notice that by mutliplication with a the only term that does not vanish in the limit is ln(x)

  • @WhiteGandalfs
    @WhiteGandalfs 7 місяців тому

    Good exercise for reforming of exponentials :D
    I was in need of some refresh in this matter!
    But that's simply not true. The limit for x > 1 is "-a" resp "-infinity" since a goes to infinity. The limit for x == 1 is 0 (resp. "undefined" if you like to define "0 * infinity" as "undefined"). The limit for x < 1 is "+infinity" - and a rapid one in that. I'm just trying to spot the point where things went wrong...
    Error in my calculation was treating b^1/e as 1/b^e - well: too long not used.

  • @vennstudios9885
    @vennstudios9885 3 місяці тому

    Ok I tried to plot this
    OH ITS NATURAL LOG
    it's the definition of the natural log

  • @akos1557
    @akos1557 7 місяців тому +1

    Yo guys, im having trouble with the homework, could someone help?
    The task : there is a regular triangle based pyramid, the height is 20 and the base edge is 18. How big is the radius of the inscribed sphere?

  • @12q8
    @12q8 10 місяців тому

    Wolframalpha says it equals log(x)

  • @damyankorena
    @damyankorena 9 місяців тому

    I wouldve exponentiated the limit to turn the subtraction into division and the L'H

  • @dhruv0x0x0
    @dhruv0x0x0 10 місяців тому

    Yaay, got it using taylor expansion

  • @ARUPBERA-ky6mq
    @ARUPBERA-ky6mq 10 місяців тому

    Sir,Actually I solved it by taking a common and then writing x^1/a as e^(lnx/a) and using expansion of the exponential part 1 and 1 get cancelled and taking the 2nd term only of the expansion we get ln x as ans.

  • @EtherTheReal
    @EtherTheReal 10 місяців тому

    Thats the dude who gets 100% on every freaking test and you cant explain why😂

  • @muddle.
    @muddle. 10 місяців тому

    bring it down down is something ive never thought of

  • @JakubS
    @JakubS 10 місяців тому

    Just use L'Hospital's rule, and you'll get the limit of x^(1/a)*lnx, which converges to lnx

  • @frederf3227
    @frederf3227 7 місяців тому

    How do you decide when the symbol means positive root and when it means all the roots? Is exchanging the positive root symbol with the power equivalent a bit sloppy?

    • @andrewclausen314
      @andrewclausen314 6 місяців тому

      The symbol always means the principle root, so positive when talking about real numbers.

  • @stevencurtis7157
    @stevencurtis7157 9 місяців тому +1

    It seems very weird to me that a function taken to a large root, multiplied by a large number, and then subtracted by a large number turns out to be the natural logarithm. I'd like to understand that particular transformation.

    • @icesandslash2839
      @icesandslash2839 9 місяців тому +1

      A large root of [not a function, but a constant, in the problem "x"] is just about 1. Multiply 1 by a large number, and you get a large number. If you compare to another large number, then indeed the questions raise: Which is larger? And can the difference be quantified?
      For many functions, the difference is in fact too large, it's said the difference "diverges", and so the limit would approach to +infinity or -infinity. However, it's the beauty of this and other calculus problems that this difference happens to converge, and those large numbers are always getting closer and closer, and their graphs end up looking like two parallel lines at distance ln(x).

  • @leob69
    @leob69 9 місяців тому

    that is such a bautiful answer

  • @antoinem4468
    @antoinem4468 9 місяців тому

    and when x = 0 ? since the function is defined from 0 to infinity due to the square root, but when you apply exp(ln(x)) there's a small problem even if the limits when x tend towards 0 of ln(x) is the same as the limit of the orignial function when a tends towards infinity. (sry for my bas english btw i'm french)

    • @methatis3013
      @methatis3013 9 місяців тому

      The limit itself does not converge for x=0. If you plug in x=0, you get the limit as a goes to infinity of -a, which is -♾

  • @daxramdac7194
    @daxramdac7194 7 місяців тому

    Nice result!

  • @ronbannon
    @ronbannon 14 днів тому

    I didn't watch the whole video, but the given problem should include a statement that x>0.

  • @kahlzun
    @kahlzun 9 місяців тому

    My math is not sufficient to understand any of this

  • @Karol-g9d
    @Karol-g9d 9 місяців тому

    Can i answer in comon ? Here goes . Lowest valid . Penultimate of 100% nothing , highest valid is penultimate of 100% something (spherically) for low infinity ? 100% nothing , for highest infinity 100% something. An ai that is eternal would likely use max 6 % total . Tomorrow morning . Nanometer amount compared to all creation . I m not sure there is enough zero after dot .cuneiform tex way is likely ideal

  • @DeadJDona
    @DeadJDona 10 місяців тому

    8:16 1/∞ = 1

  • @nohaxjustxmod-sfs3984
    @nohaxjustxmod-sfs3984 10 місяців тому

    instantly recognized it as ln(x) from all my desmos shenanigans lmao

  • @danielvega-zepeda1425
    @danielvega-zepeda1425 9 місяців тому

    Couldn’t you also just not take out the a (in step 2) and instead you have:
    Infty • 1 - infty = 0?

    • @methatis3013
      @methatis3013 9 місяців тому

      This does not hold. Infinity-infinity is not 0. Example, consider the limit as x goes to infinity of 2x - x. You get a form 2×infinity - infinity. 2×infinity is still infinity though, so by this logic, you should end up with 0 again. Or another example. Limit as x goes to infinity of x - (x-2). Limit as x goes to infinity of x-2 is infinity, so using this logic, you would get infinity - infinity again, and, again, 0. But the limit is (trivially) 2

  • @DingHang04
    @DingHang04 10 місяців тому +2

    cool trick!

  • @mathisnotforthefaintofheart
    @mathisnotforthefaintofheart 10 місяців тому +1

    I graphed it in DESMOS and y = lnx came immediately to mind. So I graphed that on too and sure it looked like an overlap. Is it a proof? Of course not, but the comment section confirmed it. Now the problem would be a bit easier "accessible" is actually the roles of "a" and "x" are reversed. That's how I look at it. Essentially you then have to show that the graph approaches lna as a horizontal asymptote.

  • @cara-seyun
    @cara-seyun 10 місяців тому +3

    x = 0 lim = -♾️
    x = 1 lim = 0
    x

  • @drakootherat1735
    @drakootherat1735 9 місяців тому

    There was a way to solve this in 4 calculations

  • @Ocklepod
    @Ocklepod 10 місяців тому

    this is very misleading, when i started doing it in my head i accidently differentiated d/dx x^(1/a) instead of d/da and the result was in the form lim y = lim -y => lim y=0

  • @ankurmondal3220
    @ankurmondal3220 9 місяців тому

    people becoming overly reliant on L' Hopital's rule is why they dont see the beauty in doing maths

  • @rikhalder5708
    @rikhalder5708 10 місяців тому

    How prove the series 1/2^√n is convergent 😢😅

  • @General12th
    @General12th 10 місяців тому

    So good!

  • @rileymack1489
    @rileymack1489 10 місяців тому

    i got this question on a quiz literally 3 hours ago
    i got it wrong
    goddamnit

  • @marekgnutek251
    @marekgnutek251 8 місяців тому

    nice!

  • @Oscaragious
    @Oscaragious 10 місяців тому

    Is there a more intuitive reason why infinity times x is problematic? Naively, it looks like it should be 0, but I can't instinctively tell you why that's not true.

  • @eralpe9965
    @eralpe9965 9 місяців тому

    felix

  • @tgx3529
    @tgx3529 10 місяців тому

    X>0

  • @DavidMartinez-ht8pp
    @DavidMartinez-ht8pp 10 місяців тому

    X^0 is not always 1, if x=0 (exactly 0) then 0^p with p tending to inf equals to 0.

  • @lollertoaster
    @lollertoaster 9 місяців тому

    Isn't this too complicated? Solving the root itself will have determinate answer so you can plug it in back to equation.
    For x > 1 limit is 0 and for 0 < x < 1, it's -a.

  • @tryndamereagiota8539
    @tryndamereagiota8539 10 місяців тому

    i guess it converges to 0

  • @swenji9113
    @swenji9113 10 місяців тому

    Using l'Hopital is so cringe

  • @ItsaDigitalHamster
    @ItsaDigitalHamster 10 місяців тому +254

    This is actually related to the famous expression for e^x:
    e^x = lim (a -> inf) [ (1 + x/a) ^ a ]
    If you ignore the limit symbol, and rearrange, you get that ln(x) is equal to the formula in the video.

    • @SimonClarkstone
      @SimonClarkstone 10 місяців тому +23

      My first thought too was "that looks like that e^x limit but turned inside-out".

  • @givrally7634
    @givrally7634 10 місяців тому +19

    Fun solution : If we have f(x) = lim a(x^1/a - 1), then
    f(x^n) = lim a((x^1/a)^n - 1^n)
    = lim a(x^1/a - 1) * (x^(n-1)/a + x^(n-2)/a + x^(n-3)/a... + 1)
    = f(x) * (1+1+1... n times)
    = n * f(x).
    So we know that f(x^n) = n f(x).
    If we plug in x = e, we get f(e) = lim a * (e^(1/a) - 1)
    take the inverse of a, you get f(e) = lim (e^a - 1) / a as a goes to 0.
    So f(e)=1.
    f(x) = f(e^ln(x)) = ln(x) * f(e) = ln(x) !
    I haven't found a way to do it using ln(ab) = ln(a) + ln(b), if anyone has an idea let me know.

    • @03abhadilipdas94
      @03abhadilipdas94 10 місяців тому

      Wrong as stated, you proved f(nx)=nf(x) for naturals “n”. Though pretty sure it’s fixable by proving for all rationals (which follows easily), then using continuity and proving it for all reals

  • @eliosedrata
    @eliosedrata 9 місяців тому +17

    It is really simple with the series expansion of exp on 0 :
    exp(lnx/a)= 1 + lnx/a + o(1/a) when a goes to infinity.

  • @StormForthcoming
    @StormForthcoming 9 місяців тому +10

    Holy crap I’ve finally learned enough math for these types of videos to actually make sense. Really cool, thank you!

  • @TranquilSeaOfMath
    @TranquilSeaOfMath 10 місяців тому +15

    Great teaching! You explained the process really well. Easy to follow. I hope the commenter is able to benefit.

  • @brianmccormick8328
    @brianmccormick8328 9 місяців тому +2

    Always funny that 0 times anything is 0. Except 0 times infinity isn’t 0. The indeterminate fight is weird.

  • @atrus3823
    @atrus3823 10 місяців тому +10

    Though it may seem weird to have a be the variable, it actually makes sense to write it this way if you know where this formula comes from. I'd say more commonly it would be n, but it doesn't really matter. This formula can be derived by thinking about how to create what is basically a continuous version of a log table. In this context, x is the variable of the function we're trying to find (ln x) and a is basically how many rectangles to use to approximate the area under the derivative of ln x, 1/x. As the number of rectangles approaches infinity, we get ln x, somewhat like a Riemann sum.

  • @VivBrodock
    @VivBrodock 9 місяців тому +5

    oh hey a fellow student brought this up in my calc class yesterday. And yeah, L'Hopital's was the method we used to solve it there. Truly the most OP tool for solving weird limits.

  • @aranarus
    @aranarus 9 місяців тому +2

    Очень сложное решение. Я просто взял производную от этого предела, получилось 1/x. Интеграл от 1/х - это ln(x)+C. Взял точку x=1, получилось, что y(1)=a(1^(1/a)-1)=0, а ln(1) тоже равен нулю, а значит С=0.

  • @GemAppleTom
    @GemAppleTom 9 місяців тому +1

    Ok… I don’t see that at all and just got 0.
    (A really big nth root goes towards 1 so you get inf.1 - inf = 0 - no 0/0 that i could see to cause a problem…)
    Why did you take the derivative? I’m guessing I’m missing something but i’m not sure what…

    • @ryalloric1088
      @ryalloric1088 9 місяців тому +1

      Yeah, that was my answer too. I don't know very much calculus, so a lot of the video went over my head, but if you don't factor out the a shouldn't it just become a - a, which is just 0?

  • @Darkstar2342
    @Darkstar2342 10 місяців тому +1

    Every time I see a Mathematician write something like "1 / ∞", something inside of me dies....

  • @notohkae
    @notohkae 9 місяців тому +1

    Why does 0/0 not equal zero? I'm confused why u need the hospital thing

    • @ragingfirefrog
      @ragingfirefrog 9 місяців тому

      Anything divided by zero is undefined. You can't calculate it. For 0/0, you can calculate what it approaches but not the actual value.

  • @S1nwar
    @S1nwar 6 місяців тому +1

    Its so amazing that this thing has a nice finite result, when at first glance it could easily diverge

  • @Skandalos
    @Skandalos 6 місяців тому

    lim of (x^(1/a) - 1)/(1/a) for a->infinity can be rewritten as limit of (x^a - 1)/a for a->0, which again can be rewritten as limit of (x^a - x^0)/(a-0) which simply is the derivative of the function f(a) = x^a at a = 0. f'(a) = ln(x)*x^a, so f'(0) = ln(x)*x^0 = ln(x)

  • @lucarionite1785
    @lucarionite1785 9 місяців тому +1

    The fact I got this right with a calc final in 2 days 🙏

  • @lowlevell0ser25
    @lowlevell0ser25 10 місяців тому +2

    I remember doing something like this but in reverse where I tried to get something to be the derivative x^-1 in polynomial form and got the same lim a-> ∞ a * x^(1/a) - a
    the process was trying to get a zero as the exponent while still getting a rate of change (1/∞) and getting 1 as a coefficent (a * 1/a) when I graphed it I realized I had to add - a to account for the infinite height I imagine this is what his friend did

    • @frantisekjanecek1641
      @frantisekjanecek1641 9 місяців тому

      I also! I needed it to approximate a logarithm on a calculator with only +, - and sqrt.

  • @Romeo-qk8tk
    @Romeo-qk8tk 10 місяців тому +1

    Luckily, ln(x) is a constant when deriving with d/da.... I'm afraid to imagine Multi-Variable Calculus 💀

  • @NovaDenizen
    @NovaDenizen 10 місяців тому +1

    Expand the Taylor series of e^((ln x)/a) and it easily works out to ln x + O(1/a)..

  • @kevinvanhorn2193
    @kevinvanhorn2193 10 місяців тому +1

    Just use x^(1/a) = exp(ln(x)/a) and then use the Taylor series expansion for exp().

  • @patsk8872
    @patsk8872 6 місяців тому

    I get a different result, can someone tell me where I went wrong? 1.) Assign the limiting value = "L" 2.) Apply ln to each side, so ln(L) = ln(the given limit) 3.) Use the rule to transpose ln and lim 4.) For the ln of the limit expression factor out a: = lim(ln[a(x^(1/a) - 1)] 5.) Apply product rule for logs = lim(ln(a) + ln(x^(1/a) - 1)) 6.) Examine the case where x > 1. We can demonstrate the difference (x^(1/a) - 1) is >= 0. Therefore we can use a comparison test, confident our limit is greater than or equal to the simpler limit, lim(ln(a)). 7.) ln(L) >= lim(ln(a)) finally evaluating, as a->infinity the limit of L also goes to infinity.

  • @modok_ff
    @modok_ff 10 місяців тому +1

    wow i actually solved it on my own!!

  • @lukandrate9866
    @lukandrate9866 8 місяців тому

    By using LH you already assume the answer, it's like calculating the limit of sin(x)/x by L'Hôpital's

  • @صليعلىرسولالله-ع6ب
    @صليعلىرسولالله-ع6ب 10 місяців тому

    Hi guys , i am a calculus student and i found some difficulties in found some hard and good exercice , so if anyone can help me by sending some website or exchange exercices ,
    Thx🙏

  • @ZebraUnite
    @ZebraUnite 10 місяців тому +8

    The highest math course I completed was a 200 level psychological stats class and I’m sitting here nodding my head like I know what the hell is going on for 8 minutes

    • @johndalton4559
      @johndalton4559 9 місяців тому +1

      this is technically high school stuff but it's true , most students even if studying chemistry or biology have no clue
      about this stuff.

    • @No-cg9kj
      @No-cg9kj 9 місяців тому

      @@johndalton4559 I'm taking honors calculus and have the only A lol. It's my 2nd week of winter break and I already have about half of the homework done for next semester lol. Most humans are terrible at math.

    • @johndalton4559
      @johndalton4559 9 місяців тому

      who asked son?@@No-cg9kj

  • @567secret
    @567secret 10 місяців тому +2

    I actually discovered this limit backwards by considering the integral of the limit as you approach 1/x. Which yields the same or something similar, so I actually had a suspicion of where this was going.

    • @AbhaySingh-ts9gt
      @AbhaySingh-ts9gt 10 місяців тому

      bro can u plz tell about this method more , i never had a thought like this , i would love to kno about it

    • @iantaakalla8180
      @iantaakalla8180 10 місяців тому +1

      It is noted that lim a -> infinity ((x^(1/a) - 1) / (1/a)) looks very much like an integral, one going from 1/a to the value that gets to 1.
      (The definition would be lim a -> 0 (F(x) - F(a)/ (x - a)) ). Therefore, one can try to pretend it is an integral to see where this limit goes.
      The reason that this works is because you have stumbled upon the definition of an integral, so one can treat it as an integral problem as opposed to a limit-solving problem.

  • @pollmacky
    @pollmacky 2 місяці тому

    Cmon man, just use Taylor series

  • @computer_geek
    @computer_geek 10 місяців тому

    I like your funny words, magic man

  • @lugyd1xdone195
    @lugyd1xdone195 9 місяців тому

    Only problem it doesnt work for x=0

  • @manny2092
    @manny2092 10 місяців тому

    I think I followed "most" of that...

  • @ajb16384
    @ajb16384 10 місяців тому

    I solved it myself and its ln(x)

  • @hadjejdjiu567
    @hadjejdjiu567 9 місяців тому

    actually that has not ( ). so..

  • @asterecho2119
    @asterecho2119 9 місяців тому

    can we say infinty0=0/0

  • @burnrise702
    @burnrise702 10 місяців тому +1

    What if the x equals to 0? You can plug in the original formula , but not for the answer . Or should we add a restriction to the answer like when x is not equals to 0 ?

    • @moih.g4570
      @moih.g4570 10 місяців тому +1

      If x=0 you factorize, and realize that a(sqrt(0)-1) just aproaches to minus infinity (I know is not sqrt, just to simplify)

    • @methatis3013
      @methatis3013 9 місяців тому

      If you plug in 0, you just get the limit as a goes to infinity of -a, which does not converge (or rather, converges to -♾)

  • @maximus111
    @maximus111 10 місяців тому

    Never liked nerds

  • @Originalimoc
    @Originalimoc 9 місяців тому

    Surprised e here

  • @soanvig
    @soanvig 8 місяців тому

    I hate limits

  • @whamer100
    @whamer100 9 місяців тому

    after just finishing calculus 1, i love how this just makes complete sense

  • @horowirtz9415
    @horowirtz9415 9 місяців тому

    e^x - 1 is equivalent to x when x is close to 0, so e^ln(x)/a - 1 ~ ln(x)/a, thus all of this equals ln(x) it's not really that hard

    • @rhum_1802
      @rhum_1802 9 місяців тому

      thanks to Landau, this exercise can be solved in 4 lines

  • @nanamacapagal8342
    @nanamacapagal8342 9 місяців тому

    You can actually solve this without L'Hopital! (as pointed out by several commenters)
    By the time you bring the a double down, just substitute h = 1/a (then h -> 0+), maybe one more substitution v = x^h - 1 (then v -> 0 and h = log_x(v + 1)).
    By now you should have you should have
    lim_(v -> 0) (v/log_x(v+1))
    You can use change of base, bring the v in the numerator double down, and use some more log properties to simplify to:
    lim_(v -> 0) (lnx/ln(1 + v)^(1/v))
    Now push the limit inside:
    lnx/ln(lim_v->0(1+v)^(1/v))
    Now that limit is the definition of e. If it still looks different, substitute ñ = 1/v. You'll see.
    On the other hand, instead of substituting v = x^h - 1, you can instead notice something about this form:
    lim_h->0 (x^h - 1)/h
    being the derivative of x^a at a = 0, derived from first principles and the definition of derivative.
    The derivative of x^a is x^a * ln(x). x^0 = 1, so what's left is ln(x).

  • @jacobstarr9010
    @jacobstarr9010 10 місяців тому

    Just plug in infinity and get -infinity 🗿

  • @ladguin3322
    @ladguin3322 10 місяців тому

    lim a->∞ a(x^1/a)-a
    lim a->∞ a.((x^1/a)-1)
    1/∞ in the limit approaches 0, therefore: lim a->∞ a.((x^0)-1)
    lim a->∞ a.(1-1)
    lim a->∞ 0
    0

  • @rockybruno2734
    @rockybruno2734 8 місяців тому

    You can also use the very powerful theorem: The Zero Bounded Theorem. Since f(x)=x and g(x)= sin(1/x) and f(0)=0 and g(x) is bounded between -1 and 1, by The Zero bounded Theorem we can conclude that lim_{x>0} xsin(1/x)=0.

  • @sailh3990
    @sailh3990 10 місяців тому

    Just use taylor expansion ... ax^1/a-a=aexp(ln(x)/a)-a but ln(x)/a goes to zero so we can say aexp(ln(x)/a)-a=a(1+ln(x)/a+o(1/a))-a=ln(x)+o(1) done.

  • @kevinluevano4746
    @kevinluevano4746 10 місяців тому

    FML I just had a final Calculus exam TODAY and I used the theorem for going to limits of sequences from limits of functions and ended up with this EXACT SAME LIMIT and I didn’t know how to follow along. And now, now, as I’m about to sleep from whatever that exam was see this video recommended.

  • @marcellomarianetti1770
    @marcellomarianetti1770 10 місяців тому

    This video is missing conditions on x, for example x^0 - 1 is not equal to 0 if x is 0 (but the case x=0 is really easy, you just plug 0 in the original limit and get -infty), and moreover the power x^1/a makes sense only if x is positive (assuming x is real)

  • @alejandromahillo3536
    @alejandromahillo3536 8 місяців тому

    I think it's easier using equivalences: x^(1/a) - 1 = e^(1/a * ln x) - 1 ~ 1/a * ln x as a -> infinity. The result follows from here

  • @anastasiaklyuch2746
    @anastasiaklyuch2746 9 місяців тому

    If you write it as a(root - a) then that is essentially "infinity*(0-1)" and that is -infinity. I'm hlad we can use easy tricks for this :)

  • @user-cg7gd5pw5b
    @user-cg7gd5pw5b 9 місяців тому

    Couldn't we simply use the Mean Value Theorem for 1/a since we have (f(t)-f(0))/(t-0) with t=1/a and f:t->x**(t)?

  • @somename5632
    @somename5632 10 місяців тому +5

    y=lim a->inf(a*x^(1/a) -a)
    Solve this for x and you get:
    x=lim a->inf(1+y/a)^a
    It follows x=e^y and thus y=ln(x).
    I did not watch the video, but what I see on the board seems more complicated.

    • @harambesson1098
      @harambesson1098 10 місяців тому

      Nice solution

    • @grivza
      @grivza 10 місяців тому

      Firstly you can't solve for x in that way. Secondly it's (a+y/a)^a, not (1+y/a)^a. And thirdly the limit(a+y/a)^a is literally as much work as the initial one.

    • @somename5632
      @somename5632 10 місяців тому

      1. Explain why.
      2. You need to divide the whole term by a not just y.
      3. Is not as much work, because its a known limit. Everybody knows it converges to e^y.

    • @grivza
      @grivza 9 місяців тому

      @@somename5632 1) Cause x is mingled with "a"s which are part of the limit, there is literally no rule to handle that. You can define a function for the term inside the limit, but then you can't get an equivelence about the limit in this way.
      2) That's correct
      3) First time I hear about that

    • @methatis3013
      @methatis3013 9 місяців тому

      ​@@grivza1) well in this case it does hold. Generally, it holds for continuous functions, so you would need to show that the function at hand is continuous.
      3) e is often defined as the limit as n goes to infinity of
      (1+1/n)^n

  • @juanestebanpineda2082
    @juanestebanpineda2082 8 місяців тому

    .

  • @munteanucatalin9833
    @munteanucatalin9833 9 місяців тому

    8 minutes for a simple limit an 11th grader can solve :P

    • @Shabudana
      @Shabudana 9 місяців тому

      Can you pls help me on 6:27 i just can't understand how the diff of e^1/aln(x) equals e^1/aln(x)ln(x)(-1/a)...Shouldn't it be only e^1/aln(x) since diff(e^α) equals e^α. ?